A particle is projected with velocity u from the bottom of an inclined plane whose inclination with the horizontal is . If afterwards the projectile strikes the inclined plane perpendicular to the surface, find the height of the point struck (distance from the ground). The angle made by the velocity with the incline is .
I got an answer while solving this question that differs from the one in my textbook. I got the correct value for the time period though...could someone please try this question? Please help me find the height and also tell me the expression you get for the time taken. Thank you! :D
Go to the website http://physics.stackexchange.com/questions/62647/projectile-along-incline for the working I did.
Easy Math Editor
This discussion board is a place to discuss our Daily Challenges and the math and science related to those challenges. Explanations are more than just a solution — they should explain the steps and thinking strategies that you used to obtain the solution. Comments should further the discussion of math and science.
When posting on Brilliant:
*italics*
or_italics_
**bold**
or__bold__
paragraph 1
paragraph 2
[example link](https://brilliant.org)
> This is a quote
\(
...\)
or\[
...\]
to ensure proper formatting.2 \times 3
2^{34}
a_{i-1}
\frac{2}{3}
\sqrt{2}
\sum_{i=1}^3
\sin \theta
\boxed{123}
Comments
Wait. Shouldn't time period be ucos(theta)/gsin(theta)? How did you get your answer?
Log in to reply
Aditya, check the image uploaded on Physics.SE. The link is above, in the discussion. My method differs from the conventional method of vector components along the incline, as I have mentioned on Physics.SE. However, the answer obtained is correct and the same in both cases. There is no doubt about the time period. Btw, check your expression...what do you mean by theta? I have not supplied such an angle...:D
FIITJEE? Cool.
Log in to reply
Yep...I recently joined FIITJEE for IIT coaching. Have you joined any coaching classes for IITJEE/Medical Entrances?
Log in to reply
BASE IIT-JEE achiever programme
Log in to reply
The time period is t=ucos(α)/gsin(β),you're changing v for u in the equation that you did.
Log in to reply
This is also a correct expression for the time period. In fact, both expressions have been equated in my textbook. But now what I need to know how to find is the height.
Log in to reply
i see,i took a rapid view at the question... you tried to use the relation between the angles? your answer may be right,just needing some trigonometry.